Intersection of Polynomial and Exponential Functions

Click For Summary
The graphs of y=x^{12} and y=2^{x} intersect at three points in the xy-plane. The polynomial function is always non-negative and increases steeply outside the interval of -1 to 1, while the exponential function also increases but eventually overtakes the polynomial. A sketch of both graphs reveals two solutions near the origin (one negative and one positive) and a third solution at approximately x=75. Using tools like Wolfram|Alpha can help confirm these intersections. The discussion emphasizes the importance of visualizing the graphs to understand their behavior and intersections.
darkchild
Messages
153
Reaction score
0

Homework Statement


At how many points in the xy-plane do the graphs of y=x^{12} and y=2^{x} intersect?


Homework Equations


none


The Attempt at a Solution


I have no idea what to do. I thought of trying to narrow it down to some intervals where the graphs may cross, but, since they're both always non-negative and always increasing, I can't see how I would do that. I also thought about approximating with Newton's Method, but that could take a while. This is a practice problem from the gre math subject test, so the method to solve it should be something that is relatively quick.
 
Physics news on Phys.org
Just draw a sketch of the two graphs. Take care to draw x^12 quite close to zero for -1 < x < 1 and draw it quite steeply increasing outside of that range (basically, don't just draw a parabola shape). Then draw 2^x. Can you see why there should be two solutions near the origin, 1 negative and another positive, and why there should be a third solution some place far away ( turns out, near x=75 ) when the exponential curve finally catches up to the polynomial, and then leaves it behind for good?
 
Question: A clock's minute hand has length 4 and its hour hand has length 3. What is the distance between the tips at the moment when it is increasing most rapidly?(Putnam Exam Question) Answer: Making assumption that both the hands moves at constant angular velocities, the answer is ## \sqrt{7} .## But don't you think this assumption is somewhat doubtful and wrong?

Similar threads

  • · Replies 6 ·
Replies
6
Views
1K
  • · Replies 3 ·
Replies
3
Views
1K
Replies
7
Views
1K
  • · Replies 12 ·
Replies
12
Views
4K
  • · Replies 15 ·
Replies
15
Views
2K
  • · Replies 8 ·
Replies
8
Views
2K
  • · Replies 17 ·
Replies
17
Views
3K
  • · Replies 2 ·
Replies
2
Views
2K
  • · Replies 12 ·
Replies
12
Views
3K
  • · Replies 3 ·
Replies
3
Views
3K